Let $ a_1, a_2,\ldots, a_n$ be non-negative real numbers. Prove that $\frac{1}{1+ a_1}+\frac{ a_1}{(1+ a_1)(1+ a_2)}+\frac{ a_1 a_2}{(1+ a_1)(1+ a_2)(1+ a_3)}+$ $\cdots+\frac{ a_1 a_2\cdots a_{n-1}}{(1+ a_1)(1+ a_2)\cdots (1+ a_n)} \le 1.$
Problem
Source: Chinese Girls Mathematical Olympiad 2012, Problem 1
Tags: inequalities, induction, inequalities proposed, Hi
10.08.2012 14:14
sqing wrote: Let $ a_1, a_2,..., a_n $ be non-negetive numbers , prove that \[\frac{1}{1+ a_1}+\frac{ a_1}{(1+ a_1)(1+ a_2)}+\frac{ a_1 a_2}{(1+ a_1)(1+ a_2)(1+ a_3)}+...\] \[+\frac{ a_1 a_2... a_{n-1}}{(1+ a_1)(1+ a_2)...(1+ a_n)} \le 1\] $ 1-\frac{1}{1+a_1}=\frac{a_1}{1+a_1};1-\frac{1}{1+a_1}-\frac{a_1}{(1+a_1)(1+a_2)}=\frac{a_1a_2}{(1+a_1)(1+a_2)}$ ... $1-(\frac{1}{1+ a_1}+\frac{ a_1}{(1+ a_1)(1+ a_2)}+\frac{ a_1 a_2}{(1+ a_1)(1+ a_2)(1+ a_3)}+... +\frac{ a_1 a_2... a_{n-1}}{(1+ a_1)(1+ a_2)...(1+ a_n)})=\frac{a_1a_2...a_n}{(1+a_1)(1+a_2)...(1+a_n)}\ge0 $ The equality holds iff $a_1=0$ or $a_2=0$ or...or $a_n=0$
10.08.2012 14:17
sqing wrote: Let $ a_1, a_2,..., a_n $ be non-negetive numbers , prove that \[\frac{1}{1+ a_1}+\frac{ a_1}{(1+ a_1)(1+ a_2)}+\frac{ a_1 a_2}{(1+ a_1)(1+ a_2)(1+ a_3)}+...\] \[+\frac{ a_1 a_2... a_{n-1}}{(1+ a_1)(1+ a_2)...(1+ a_n)} \le 1\] \[\frac{1}{1+ a_1}+\frac{ a_1}{(1+ a_1)(1+ a_2)}+\frac{ a_1 a_2}{(1+ a_1)(1+ a_2)(1+ a_3)}+...\] \[+\frac{ a_1 a_2... a_{n-1}}{(1+ a_1)(1+ a_2)...(1+ a_n)} \] \[\le \frac{1}{1+ a_1}+\frac{ a_1}{(1+ a_1)(1+ a_2)}+\frac{ a_1 a_2}{(1+ a_1)(1+ a_2)(1+ a_3)}+...\] \[+\frac{ a_1 a_2... a_{n-1}}{(1+ a_1)(1+ a_2)...(1+ a_{n-1})}=1 \] Equality holds when $a_n=0$
10.08.2012 14:24
Proof: \[\frac{1}{1+{{a}_{1}}}+\frac{{{a}_{1}}}{(1+{{a}_{1}})(1+{{a}_{2}})}+\frac{{{a}_{1}}{{a}_{2}}}{(1+{{a}_{1}})(1+{{a}_{2}})(1+{{a}_{3}})}+...+\frac{{{a}_{1}}{{a}_{2}}...{{a}_{n-1}}}{(1+{{a}_{1}})(1+{{a}_{2}})...(1+{{a}_{n}})}=\] \[\frac{1}{1+{{a}_{1}}}+\sum\limits_{k=2}^{n}{\frac{{{a}_{1}}{{a}_{2}}...{{a}_{k-1}}}{(1+{{a}_{1}})(1+{{a}_{2}})...(1+{{a}_{k}})}}=\frac{1}{1+{{a}_{1}}}+\sum\limits_{k=2}^{n}{\frac{{{a}_{1}}{{a}_{2}}...{{a}_{k-1}}\left( 1+{{a}_{k}}-{{a}_{k}} \right)}{(1+{{a}_{1}})(1+{{a}_{2}})...(1+{{a}_{k}})}}=\] \[\frac{1}{1+{{a}_{1}}}+\sum\limits_{k=2}^{n}{\left( \frac{{{a}_{1}}{{a}_{2}}...{{a}_{k-1}}}{(1+{{a}_{1}})(1+{{a}_{2}})...(1+{{a}_{k-1}})}-\frac{{{a}_{1}}{{a}_{2}}...{{a}_{k}}}{(1+{{a}_{1}})(1+{{a}_{2}})...(1+{{a}_{k}})} \right)}=\] \[\frac{1}{1+{{a}_{1}}}+\frac{{{a}_{1}}}{1+{{a}_{1}}}-\frac{{{a}_{1}}{{a}_{2}}...{{a}_{n}}}{(1+{{a}_{1}})(1+{{a}_{2}})...(1+{{a}_{n}})}\le 1.\]
10.08.2012 14:40
Generalization Let $a_1, a_2,..., a_n$ be non-negative numbers and $f:\left[ 0,\infty \right)\to \left[ 0,\infty \right),g:\left[ 0,\infty \right)\to \left( 0,\infty \right)$, prove that \[\frac{g\left( {{a}_{1}} \right)-f\left( {{a}_{1}} \right)}{g\left( {{a}_{1}} \right)}+\frac{f\left( {{a}_{1}} \right)}{g\left( {{a}_{1}} \right)g\left( {{a}_{2}} \right)}\left( g\left( {{a}_{2}} \right)-f\left( {{a}_{2}} \right) \right)+\frac{f\left( {{a}_{1}} \right)f\left( {{a}_{2}} \right)}{g\left( {{a}_{1}} \right)g\left( {{a}_{2}} \right)g\left( {{a}_{3}} \right)}\left( g\left( {{a}_{3}} \right)-f\left( {{a}_{3}} \right) \right)+...\] \[+\frac{f\left( {{a}_{1}} \right)f\left( {{a}_{2}} \right)...f\left( {{a}_{n-1}} \right)}{g\left( {{a}_{1}} \right)g\left( {{a}_{2}} \right)...g\left( {{a}_{n}} \right)}\left( g\left( {{a}_{n}} \right)-f\left( {{a}_{n}} \right) \right)\le 1.\]
11.08.2012 01:36
(Short list IMO,1998) Let $ a_1, a_2,..., a_n $ be positive numbers with $ a_1+a_2+...+a_n<1 $, prove that \[\frac{ a_1 a_2... a_n}{(1- a_1)(1- a_2)...(1- a_n)} \le \frac{a_1+a_2+...+a_n}{n^{n+1}(1-(a_1+a_2+...+a_n))}.\]
11.08.2012 02:43
sqing wrote: (Short list IMO,1998) Let $ a_1, a_2,..., a_n $ be positive numbers with $ a_1+a_2+...+a_n<1 $, prove that \[\frac{ a_1 a_2... a_n}{(1- a_1)(1- a_2)...(1- a_n)} \le \frac{a_1+a_2+...+a_n}{n^{n+1}(1-(a_1+a_2+...+a_n))}.\] $a_{n+1}=1-\sum_{i=1}^{n}a_i;a_{n+1}>0$ The desired inequality becomes $\frac{ a_1 a_2... a_na_{n+1}}{(1- a_1)(1- a_2)...(1- a_n)(1- a_{n+1})} \le \frac{1}{n^{n+1}}$ $1-a_i=a_1+...+a_{i-1}+a_{i+1}+...+a_{n+1}\ge n\sqrt[n]{a_1...a_{i-1}a_{i+1}...a_{n+1}}$ Done.
14.08.2012 08:19
I think the problem is related to Probability. See here. http://www.artofproblemsolving.com/Forum/viewtopic.php?f=498&t=493730
15.08.2012 17:07
We claim \[ \frac {1}{a_1+1}+\frac {a_1}{(1+a_1)(1+a_2)}+.....+\frac {a_1a_2.....a_{n-1}}{(1+a_1)(1+a_2).....(1+a_n)}\] \[=1-\frac{a_1a_2.....a_n}{(1+a_1)(1+a_2).....(1+a_n)} \] We'll prove this using mathematical induction. For $n=1,(1)$ is true since $\frac {1}{1+a_1}=1-\frac {a_1}{1+a_1}$ Let $(1)$ be true for some $m \in \mathbb {N}$ So \[\frac {1}{1+a_1}+\frac {a_1}{(a_1+1)(a_2+1)}+.....+\frac {a_1a_2.....a_{m-1}}{(1+a_1)(1+a_2).....(1+a_m)}\] \[=1-\frac {a_1a_2.....a_m}{(1+a_1)(1+a_2).....(1+a_m)}\] Now for $m+1$, \[\frac {1}{1+a_1}+\frac {a_1}{(a_1+1)(a_2+1)}+.....+\frac {a_1a_2.....a_{m-1}}{(1+a_1)(1+a_2).....(1+a_m)}+\frac {a_1a_2.....a_m}{(1+a_1)(1+a_2).....(1+a_{m+1})}\] \[=1-\frac {a_1a_2.....a_m}{(1+a_1)(1+a_2).....(1+a_m)}+\frac {a_1a_2.....a_m}{(1+a_1)(1+a_2).....(1+a_{m+1})}\] \[=1-\frac {a_1a_2.....a_{m+1}}{(1+a_1)(1+a_2).....(1+a_{m+1})}\] So by mathematical induction $(1)$ holds $\forall n \in \mathbb {N}$. From $(1)$ we get $\frac {1}{a_1+1}+\frac {a_1}{(1+a_1)(1+a_2)}+.....+\frac {a_1a_2.....a_{n-1}}{(1+a_1)(1+a_2).....(1+a_n)} \le 1$ because $\frac{a_1a_2.....a_n}{(1+a_1)(1+a_2).....(1+a_n)} \ge 0$ as $a_1,a_2,.....,a_n$ are non-negative with equality iff at least one of $a_1,a_2,.....,a_n$ is equal to $0$
18.08.2012 15:19
Solution: Let $b_1=(1+a_2)...(1+a_n), b_n=a_1a_2...a_{n-1}, b_{n+1}=a_1a_2...a_{n}$ and $b_i=a_1a_2...a_{i-1}(1+a_{i+1})...(1+a_n)$ for $2\leq i\leq n-1$.Furthermore let $B=(1+a_1)(1+a_2)...(1+a_n)$.The inequality can be re-written as \[A=\sum _{k=1}^n b_k\leq B\]Now notice that \[A+b_{n+1}=\sum _{k=1}^{n-1} b_k+(1+a_n)b_n=\sum _{k=1}^{n-2} b_k+a_1a_2...a_{n-2}(1+a_n)+(1+a_n)b_{n}=\sum _{k=1}^{n-2} b_k+(1+a_{n-1})b_{n-1}\] By continuing this argument at the last we get $A+b_{n+1}=(1+a_1)b_1=B$ and so we are done.The equality occurs if $b_{n+1}=0$ i.e. some of the $a_i$s is equal to zero.
31.08.2012 02:23
Solution by Kuronyanko, Japan. Let $Q_n=(1+a_1)(1+a_2)\ \cdots (1+a_n)$ $P_n=a_1a_2\cdots a_n$ $Q_0=1,\ P_0=1$. We can rephrase the given inequality as \[\frac{P_0}{Q_1}+\frac{P_1}{Q_2}+\cdots +\frac{P_{n-1}}{Q_n}\leq 1\ \cdots [*]\] Proof : $1-\frac{P_0}{Q_1}=\frac{a_1P_0}{Q_1}=\frac{P_1}{Q_1}$, $\frac{P_1}{Q_1}-\frac{P_1}{Q_2}=\frac{a_2P_1}{Q_2}=\frac{P_2}{Q_2}.$ Generally, $\frac{P_i}{Q_i}-\frac{P_i}{Q_{i+1}}=\frac{a_{i+1}P_i}{Q_{i+1}}=\frac{P_{i+1}}{Q_{i+1}}.$ Summing up, from $i=1$ through $n-1$, yielding $1-\sum_{i=0}^{n-1} \frac{P_i}{Q_{i+1}}=\frac{P_n}{Q_n}\geq 0.$ Equality holds when $a_i=0$ for some $i$.
13.10.2012 05:29
You could take a a1/(a1+1) out of the last n-1 terms and use induction.
12.05.2014 09:54
Let $a_1,a_2,\dots,a_n$ be positive real numbers whose product is $1$. Show that \[ \frac{a_1}{1+a_1}+\frac{a_2}{(1+a_1)(1+a_2)}+...+\frac{a_n}{(1+a_1)(1+a_2)...(1+a_n)} \ge \frac{2^n-1}{2^n} .\] 2014 CMO
29.05.2014 20:02
$\frac{a_1}{1+a_1}+.......+\frac{a-n}{(1+a_1)....(1+a_n)}\$ $= 1-(1/1+a_1)+......+ (1/(1+a_1)....(1+a_n-1)- (1/(1+a_1)....(1+a_n))$ hence,$\[ \frac{a_1}{1+a_1}+\frac{a_2}{(1+a_1)(1+a_2)}+...+\frac{a_n}{(1+a_1)(1+a_2)...(1+a_n)}$ = $1- 1/(1+a_1)....(1+a_n)$ now to prove that $\[ \frac{a_1}{1+a_1}+\frac{a_2}{(1+a_1)(1+a_2)}+...+\frac{a_n}{(1+a_1)(1+a_2)...(1+a_n)}\ge\frac{2^n-1}{2^n}. \]$ it reduces to prove $1- 1/(1+a_1)....(1+a_n)ge\frac{2^n-1}{2^n}. $ now this reduces to proving $1/(1+a_1)...(1+a_n) <= (1/2^n)$ or $(1+a_1)(1+a_2)....(1+a_n) >= 2^n$ which easily follows by AM $>=$ GM since $1+a1 >= 2 a_1^1/2$ ..... $1+a_n >= 2xa_n^1/2$ hence proved!
07.02.2015 22:59
Note that $a_n$ only appears in the denominator, so we can assume it is 0 (to get a maximum). $ \frac{1}{1+a_1}+\frac{ a_1}{(1+a_1)(1+a_2)}+\frac{ a_1 a_2}{(1+a_1)(1+a_2)(1+a_3)}+ $ $ \cdots+\frac{ a_1 a_2\cdots a_{n-1}}{(1+a_1)(1+a_2)\cdots (1+a_n)}$ $=\frac{1}{1+a_1}+\frac{ a_1}{(1+a_1)(1+a_2)}+\frac{ a_1 a_2}{(1+a_1)(1+a_2)(1+a_3)}+ $ $ \cdots+\frac{ a_1 a_2\cdots a_{n-3}}{(1+a_1)(1+a_2)\cdots (1+a_{n-2})}+\frac{ a_1 a_2\cdots a_{n-2}}{(1+a_1)(1+a_2)\cdots (1+a_{n-1})}+\frac{ a_1 a_2\cdots a_{n-1}}{(1+a_1)(1+a_2)\cdots (1+a_{n-1})}$ $=\frac{1}{1+a_1}+\frac{ a_1}{(1+a_1)(1+a_2)}+\frac{ a_1 a_2}{(1+a_1)(1+a_2)(1+a_3)}+ $ $ \cdots+\frac{ a_1 a_2\cdots a_{n-3}}{(1+a_1)(1+a_2)\cdots (1+a_{n-2})}+\frac{ a_1 a_2\cdots a_{n-2}}{(1+a_1)(1+a_2)\cdots (1+a_{n-2})}$ $= \cdots=\frac{1}{1+a_1}+\frac{a_1}{1+a_1}=1$, as required.
05.01.2017 14:52
Let $a_{i}=\tan^2(x_{i}), x_{i} \in (0, \pi /2)$, using $\frac{1}{1+\tan^2(x)}=\cos^2(x)$, inequality transform to $\sum_{i=1}^{n}\frac{a_{1}a_{2}\ldots s_{i-1}}{(1+a_{1})(1+a_{2})\ldots (1+a_i{})}$ = $\sum_{i=1}^{n}\sin^2(x_{1})\sin^2(x_{2})\ldots \sin^2(x_{i-1})\cos^2(x_{i})$ = $\sum_{i=1}^{n}(\sin^2(x_{1})\sin^2(x_{2})\ldots \sin^2(x_{i-1}) - \sin^2(x_{1})\sin^2(x_{2})\ldots \sin^2(x_{i-1})\sin^2(x_{i}))$ = $1 - \sin^2(x_{1})\sin^2(x_{2})\ldots \sin^2(x_{n}) \leq 1$, which is obvius.
10.03.2018 20:55
Think this solution is slightly different from others, may be wrong?
13.10.2018 01:45
This problem is absolutely hilarious.
20.03.2021 05:25
We proceed to prove this by induction. Base Case: $n=1$ $$\frac{1}{1+a_1}\leq 1\iff 1\leq 1+a_1\iff a_1\geq 0$$which is true by condition. Assume that for some positive integer $k$, we have that $$\frac{1}{1+a_1}+\cdots +\frac{a_1\cdots a_{k-1}}{(1+a_1)\cdots (1+a_k)}\leq 1$$note that this is the same as $$\frac{1}{1+a_1}(1+\frac{a_1}{1+a_2}(\cdots (1+\frac{a_{n-2}}{1+a_{n-1}}(1+\frac{a_{n-1}}{1+a_n}))\cdots))\leq 1$$Then notice that $$\frac{1}{1+a_1}+\cdots +\frac{a_1\cdots a_k}{(1+a_1)\cdots (1+a_{k+1})} $$$$= \frac{1}{1+a_1}(1+\frac{a_1}{1+a_2}(\cdots (1+\frac{a_{k-1}}{1+a_k}(1+\frac{a_k}{1+a_{k+1}}))\cdots)) \leq 1$$$$\iff 1+\frac{a_1}{1+a_2}(\cdots (1+\frac{a_{k-1}}{1+a_k}(1+\frac{a_k}{1+a_{k+1}}))\cdots)\leq 1+a_1$$$$\iff \frac{a_1}{1+a_2}(\cdots (1+\frac{a_{k-1}}{1+a_k}(1+\frac{a_k}{1+a_{k+1}}))\cdots) \leq a_1$$Then we have two cases, if $a_1 = 0$, we are obviously done with $0\leq 0$, if it's not $0$, we continue, $$\iff \frac{1}{1+a_2}(\cdots (1+\frac{a_{n-1}}{1+a_n}(1+\frac{a_n}{1+a_{n+1}}))\cdots)\leq 1$$However here, by setting $$a_1\to a_2\;\;a_2\to a_3\;\cdots\;a_{k}\to a_{k+1}$$in the inductive hypothesis, we have the desired result, and we complete our induction.
20.03.2021 12:58
06.04.2021 19:43
The main difficulty of this problem is observing that no inequality is involved. We will prove by induction that the sum is equal to $1-\frac{\prod_{i=1}^{n}a_i}{\prod_{i=1}^{n}(a_i+1)}$, which also establishes the equality case as when $\prod\limits_{i=1}^{n}a_i=0$. The base case, $n=1$, is trivial. For the inductive step, notice that $\frac{\prod_{i=1}^{n}a_i}{\prod_{i=1}^{n}(a_i+1)}=\frac{(\prod_{i=1}^{n}a_i)(a_{n+1}+1)}{\prod_{i=1}^{n+1}(a_i+1)}$ and $\frac{(\prod_{i=1}^{n}a_i)(a_{n+1}+1)}{\prod_{i=1}^{n+1}(a_i+1)}-\frac{(\prod_{i=1}^{n}a_i)}{\prod_{i=1}^{n+1}(a_i+1)}=\frac{(\prod_{i=1}^{n}a_i)(a_{n+1}+1-1)}{\prod_{i=1}^{n+1}(a_i+1)}=\frac{(\prod_{i=1}^{n+1}a_i)}{\prod_{i=1}^{n+1}(a_i+1)}$.
30.06.2022 02:01
Induct on $n$. The base case $n=1$ is trivial; for the inductive step, note that \begin{align*} \frac 1{a_1+1} + \frac{a_1}{(1+a_1)(1+a_2)}+\cdots+\frac{a_1a_2\cdots a_{n-1}}{(1+a_1)(1+a_2) \cdots (1+a_n)} &\leq 1 \\ \iff 1+\frac{a_1}{1+a_2}+\cdots+\frac{a_1a_2\cdots a_{n-1}}{(1+a_2)(1+a_3)\cdots (1+a_n)} &\leq 1+a_1 \\ \iff \frac 1{1+a_2} + \frac{a_2}{(1+a_2)(1+a_3)}+\cdots+\frac{a_2a_3\cdots a_{n-1}}{(1+a_2)(1+a_3)\cdots (1+a_n)} &\leq 1, \end{align*}which is just the case for $n-1$ variables.
09.02.2023 18:11
Let prove this inequality by induction. Let $\P(x)=\frac{a_1a_2\cdots a_{x-1}}{(1+a_1)(1+a_2)\cdots(1+a_x)}$ $\textcolor{red}{Claim:}$ For any positive integer $n$ \[ \sum_{i=1}^n \left( \P(i) \right)=1-\frac{a_1a_2\cdots a_{n+1}}{(1+a_1)(1+a_2)\cdots(1+a_{n+2})}\]For $n=1$ it is trivial: $\frac {1}{1+a_1}=1-\frac {a_1}{1+a_1}$ Assume that this is true for some $m$ $\implies$ \[ \sum_{i=1}^m \left( \P(i) \right)=1-\frac{a_1a_2\cdots a_{m+1}}{(1+a_1)(1+a_2)\cdots(1+a_{m+2})}\]Now for $m+1$, \[\frac {1}{1+a_1}+.....+\frac {a_1a_2.....a_{m+1}}{(1+a_1)(1+a_2).....(1+a_{m+2)}}+\frac {a_1a_2.....a_{m+2}}{(1+a_1)(1+a_2).....(1+a_{m+3})}\]\[=1-\frac {a_1a_2.....a_{m+2}}{(1+a_1)(1+a_2).....(1+a_{m+2})}+\frac {a_1a_2.....{a_{m+2}}}{(1+a_1)(1+a_2).....(1+a_{m+3})}\]\[=1-\frac {a_1a_2.....a_{m+3}}{(1+a_1)(1+a_2).....(1+a_{m+3})}\]Which means that \[ \sum_{i=1}^{n+1} \left( \P(i) \right)=1-\frac{a_1a_2\cdots a_{n+2}}{(1+a_1)(1+a_2)\cdots(1+a_{n+3})}\]is also true. So our claim is true for all positive integer $n$. Equality holds if $a_n = 0$.
07.08.2023 05:28
For $1 \le i \le n$, let $p_i = \tfrac{1}{1 + a_i}$, so $0 < p_i \le 1$. We now wish to prove that $$p_1 + (1 - p_1)p_2 + (1 - p_1)(1 - p_2)p_3 + \cdots + (1 - p_1)(1 - p_2) \cdots (1 - p_{n - 1})p_n \le 1.$$Now, consider $n$ coins $C_1, C_2, \ldots, C_n$ such that the probability of $C_i$ flipping heads is $p_i$. Then the probability of eventually getting heads if we first flip $C_1$, then $C_2$, and so on and stop after we flips heads is the sum in question; but obviously this is less than or equal to $1$. Equality holds when the probability of any of the coins flipping heads is $1$, i.e. at least one of the $a_i$ equals $0$.
20.08.2023 01:16
Noice. Consider adding all fractions on the LHS, and the numerator of the sum. Note that for $n = 2$ it is of the form, \[ (1+a_1) + a_1(1+a_1)(1+a_2) = (1 + a_1)(1+a_2) - a_1a_2 \]Now assume for $n = k$ we have, \begin{align*} (1+a_2)\dots(1+a_{k}) + a_1(1+a_2)\dots&(1+a_{k}) + \dots + a_1a_2\dots a_{k}\\ &= (1+a_1)(1+a_2)\dots(1+a_{k}) - a_1a_2\dots a_{k} \end{align*}Then for $n = k+1$ we wish to have, \begin{align*} (1+a_2)\dots(1+a_{k+1}) + a_1(1+a_2)\dots&(1+a_{k+1}) + \dots + a_1a_2\dots a_{k+1}\\ &= (1+a_1)(1+a_2)\dots(1+a_{k+1}) - a_1a_2\dots a_{k+1} \end{align*}The LHS can be written as, \begin{align*} (1+a_{k+1}) \cdot [(1+a_1)(1+a_2)\dots &(1+a_k) - a_1a_2\dots a_k]\\ &= (1+a_1)(1+a_2)\dots (1+a_{k+1}) - a_1a_2\dots a_{k+1} \end{align*}which completes our induction as desired. Now from our induction we have, \[ \frac{1}{1+a_1} + \frac{a_1}{(1+a_1)(1+a_2)} + \cdots + \frac{a_1a_2\cdots a_{n-1}}{(1+a_1)(1+a_2)\cdots(1+a_n)} = 1 - \frac{a_1a_2\dots a_n}{(1+a_1)(1+a_2)\dots (1+a_n)}.\] Thus the fraction is always less than 1 with equality holding if and only if some $a_i = 0$.
10.02.2024 00:23
Counting + Algebra = ???? Define $x_i = \frac{1}{1+a_i},$ so that $0 < x_i \le 1.$ The problem is then equivalent to $$x_1 + (1-x_1) x_2 + (1-x_1)(1-x_2) x_3 + \dots \le 1.$$Now, create $n$ coins, where the $i$th coin has a probability of $x_i$ to come up heads. Consider the sequence of actions of flipping all of the $n$ coins in order until we get a coin that flips heads. The above expression is then the probability that we see one of the coins come up heads, which is clearly less than or equal to $1.$ Equality holds if and only if one of the coins always comes up heads, i.e. at least one of the $x_i$ is equal to $1.$ This is equivalent to at least one of the $a_i$ being equal to $0,$ and we are done.
23.04.2024 12:01
It is sufficient to show that \[1-\left(\frac{1}{1+ a_1}+\frac{ a_1}{(1+ a_1)(1+ a_2)}+\cdots+\frac{ a_1 a_2\cdots a_{n-1}}{(1+ a_1)(1+ a_2)\cdots (1+ a_n)}\right)=\frac{a_1a_2\cdots a_n}{(1+a_1)(1+a_2)\cdots(1+a_n)}\]But this follows from induction on $n$ as \[\frac{a_1a_2\cdots a_n}{(1+a_1)(1+a_2)\cdots(1+a_n)}-\frac{a_1a_2\cdots a_n}{(1+a_1)(1+a_2)\cdots(1+a_n)(1+a_{n+1})}=\frac{a_1a_2\cdots a_na_{n+1}}{(1+a_1)(1+a_2)\cdots(1+a_n)(1+a_{n+1})}\]
22.06.2024 06:37
Clearly, $\tfrac{1}{1+a_n} \leq 1$. Then: \begin{align*} \dfrac{a_{n - 1}}{1 + a_n} \leq a_{n-1} \quad \longrightarrow \quad 1 + \dfrac{a_{n - 1}}{1 + a_n} \leq 1 + a_{n-1} \quad \longrightarrow \quad \dfrac{1}{1+a_{n-1}} + \dfrac{a_{n-1}}{\left(1+a_n\right)\left(1+a_{n-1}\right)} \leq 1. \end{align*}Repeating the process, we are guaranteed to obtain the final result.
24.06.2024 19:56
03.12.2024 06:02
If $S_i$ is the $i$th elementary symmetric polynomial of the roots, it is possible to show by induction that $$\text{LHS} = \frac{S_0+S_1+\cdots+S_{n-1}}{\prod_{i=1}^n (1+a_i)}.$$Therefore, we have that $$\text{LHS} = \frac{\prod_{i=1}^n (1+a_i) - S_n}{\prod_{i=1}^n (1+a_i)} = 1 - \prod_{i=1}^n \frac{a_i}{1+a_i} \leq 1,$$so the inequality is proven. From here, it is clear that equality holds if and only if at least one of the $a_i=0.$ QED
10.01.2025 07:14
why can't more ineqs be like this \[\frac{1}{1+a_1} + \frac{a_1}{(1+a_1)(1+a_2)} + \cdots + \frac{a_1a_2\cdots a_{n-1}}{(1+a_1)(1+a_2)\cdots(1+a_n)} \]\[= 1-\frac{a_1}{1+a_1} + \frac{a_1}{1+a_1}-\frac{a_1a_2}{(1+a_1)(1+a_2)} + \cdots + \frac{a_1a_2\cdots a_{n-1}}{(1+a_1)(1+a_2)\cdots(1+a_{n-1})}-\frac{a_1a_2\cdots a_{n-1}a_n}{(1+a_1)(1+a_2)\cdots(1+a_n)}\]\[= 1-\frac{a_1a_2\cdots a_{n-1}a_n}{(1+a_1)(1+a_2)\cdots(1+a_n)}.\]This is clearly $\le 1$ with equality case when any of the variables is $0$.